LSAT and Law School Admissions Forum

Get expert LSAT preparation and law school admissions advice from PowerScore Test Preparation.

User avatar
 Dave Killoran
PowerScore Staff
  • PowerScore Staff
  • Posts: 5852
  • Joined: Mar 25, 2011
|
#80547
This game is also discussed in our Podcast: LSAT Podcast Episode 70: The May 2020 LSAT-Flex Logic Games Section

Complete Question Explanation
(The complete setup for this game can be found here: viewtopic.php?t=33052)

The correct answer choice is (E).

The question stem places M on day 2, which immediately triggers the second rule, placing M on day 7 as well:


  • ___ ..... _M_ ..... ___ ..... ___ ..... ___ ..... ___ ..... _M_ ..... ___
     1 .....     2 .....    3 .....    4 .....     5 .....    6 .....      7 .....   8
With M placed, the third rule comes into effect, instantly eliminating O from being played on days 1, 3, 6, and 8. And, since O cannot be performed on day 1, via the first rule it also cannot be performed on day 5. At this point, O is incredibly restricted, and only one day is left for O: day 4. Thus, answer choice (E) is correct.


Answer choice (E): This is the correct answer choice.

Get the most out of your LSAT Prep Plus subscription.

Analyze and track your performance with our Testing and Analytics Package.